Đến nội dung

Hình ảnh

[TOPIC] ÔN THI BẤT ĐẲNG THỨC $\boxed{\text{THPT CHUYÊN}}$ LỚP $10$ năm $2018-2019$

bất đẳng thức holder cosi bunhiacopxki

  • Chủ đề bị khóa Chủ đề bị khóa
Chủ đề này có 318 trả lời

#241
Khoa Linh

Khoa Linh

    Thiếu úy

  • Thành viên
  • 601 Bài viết

105. Cho $a\,,b\,\,> 0$. Chứng minh rằng:

 

$\frac{1}{2 + a + b} + \frac{a}{2a + b + 1} + \frac{b}{2b + a + 1} \,\, \leqq \frac{3}{4}$

Anh viet9a14124869 lời giải bài này đâu cần phức tạp quá vậy ạ... 

Ta có:

$\frac{4}{a+b+2}+\frac{4a}{2a+b+1}+\frac{4b}{2b+a+1}=\frac{4}{(a+1)+(b+1)}+\frac{4a}{(a+b)+(a+1)}+\frac{4b}{(a+b)+b+1}\leq \frac{1}{a+1}+\frac{1}{b+1}+\frac{a}{a+b}+\frac{a}{a+1}+\frac{b}{a+b}+\frac{b}{b+1}=3$


$\sqrt[LOVE]{MATH}$

"If I feel unhappy, I do mathematics to become happy. If I am happy, I

 

do mathematics to keep happy" - Alfréd nyi 


#242
viet9a14124869

viet9a14124869

    Trung úy

  • Thành viên
  • 903 Bài viết

Anh viet9a14124869 lời giải bài này đâu cần phức tạp quá vậy ạ... 

Ta có:

$\frac{4}{a+b+2}+\frac{4a}{2a+b+1}+\frac{4b}{2b+a+1}=\frac{4}{(a+1)+(b+1)}+\frac{4a}{(a+b)+(a+1)}+\frac{4b}{(a+b)+b+1}\leq \frac{1}{a+1}+\frac{1}{b+1}+\frac{a}{a+b}+\frac{a}{a+1}+\frac{b}{a+b}+\frac{b}{b+1}=3$

Anh quên mất ^^ đi ngủ mới nghĩ ra cái này  :lol:  :lol: .....


                                                                    SÓNG BẮT ĐẦU TỪ GIÓ

                                                                    GIÓ BẮT ĐẦU TỪ ĐÂU ?

                                                                    ANH CŨNG KHÔNG BIẾT NỮA 

                                                                    KHI NÀO...? TA YÊU NHAU .


#243
xuanhoan23112002

xuanhoan23112002

    Trung sĩ

  • Thành viên
  • 103 Bài viết

Bài 106Cho a1, a2,...,a19 là các số tự nhiên thỏa mãn: a1+a2+...+a19 =26. Tìm giá trị nhỏ nhất của:

S=a12+a22+...+a192



#244
tr2512

tr2512

    Thượng sĩ

  • Thành viên
  • 272 Bài viết

Bài 107:

Cho 3 số thực $a, b, c$ thỏa mãn $a+b+c=1$. Chứng minh:

$\frac{a}{{{a^2} + 1}} + \frac{b}{{{b^2} + 1}} + \frac{c}{{{c^2} + 1}} \le \frac{9}{{10}}$(Poland MO 1996)


Bài viết đã được chỉnh sửa nội dung bởi tr2512: 29-04-2018 - 16:17


#245
viet9a14124869

viet9a14124869

    Trung úy

  • Thành viên
  • 903 Bài viết

Bài toán số 108 : Cho a,b,c là các số thực dương thỏa mãn a+b+c=1 . Với q=ab+bc+ca , chứng minh rằng : 

$$\frac{a}{(1-a)^2}+\frac{b}{(1-b)^2}+\frac{c}{(1-c)^2}\geq \frac{1}{2q(1-q)}$$

                                                                                                                                                              ____ Nguyễn Đức Việt ____

 

P/S : Yếu quá :P ....


Bài viết đã được chỉnh sửa nội dung bởi viet9a14124869: 29-04-2018 - 08:50

                                                                    SÓNG BẮT ĐẦU TỪ GIÓ

                                                                    GIÓ BẮT ĐẦU TỪ ĐÂU ?

                                                                    ANH CŨNG KHÔNG BIẾT NỮA 

                                                                    KHI NÀO...? TA YÊU NHAU .


#246
yeutoan89

yeutoan89

    Binh nhì

  • Thành viên mới
  • 15 Bài viết

Bài 109: cho x,y là các số thực dương. Tìm Min của

$\fn_phv \frac{xy}{x^{2}+y^{2}}+\left ( \frac{1}{x} +\frac{1}{y}\right )\sqrt{2.(x^{2})+y^{2}}$

Dấu "=" xảy ra khi nào

mọi người giải giúp nhé. mÌnh sửa rồi mà ko xóa được \fnp

h

v


Bài viết đã được chỉnh sửa nội dung bởi yeutoan89: 29-04-2018 - 14:50


#247
yeutoan89

yeutoan89

    Binh nhì

  • Thành viên mới
  • 15 Bài viết

Bài 109: cho x,y là các số thực dương. Tìm MIn

$\fn_phv \frac{xy}{x^{2}+y^{2}}+(\frac{1}{x}+\frac{1}{y}).{\sqrt{2.(x^{2}+y^{2})}}$

dấu "=" xảy ra khi nào


Bài viết đã được chỉnh sửa nội dung bởi yeutoan89: 29-04-2018 - 14:43


#248
DOTOANNANG

DOTOANNANG

    Đại úy

  • ĐHV Toán Cao cấp
  • 1609 Bài viết

105. Cho $a\,,b\,\,> 0$. Chứng minh rằng:

 

$\frac{1}{2 + a + b} + \frac{a}{2\,a + b + 1} + \frac{b}{2\,b + a + 1} \,\, \leqq \frac{3}{4}$

 

Đẳng thức có khi: $a= b= 1$

 

Do vậy, ta viết lại bất đẳng thức dưới dạng:

 

$\frac{1}{2 + a + b} -\frac{1}{2}+ \frac{a}{2\,a + b + 1}-\frac{1}{2} + \frac{b}{2\,b + a + 1} -\frac{1}{2}\leqq \frac{3}{4}-\frac{3}{2}$

 

hay:

 
$\frac{a+b}{2 + a + b} + \frac{b+1}{2\,a + b + 1} + \frac{a+1}{2\,b + a + 1} \geqq \frac{3}{2}$
 
Ta có:
 
$\frac{a+b}{2 + a + b} + \frac{b+1}{2\,a + b + 1} + \frac{a+1}{2\,b + a + 1}=$
 
$=\frac{(a+b)^2}{(a+b)\,(2 + a + b)} + \frac{(b+1)^2}{(b+1)\,(2\,a + b + 1)} + \frac{(a+1)^2}{(a+1)\,(2\,b + a + 1)}\geqq$
 
$\geqq\frac{(a+b+a+1+b+1)^2}{(a+b)\,(2 + a + b)+(b+1)\,(2\,a + b + 1)+(a+1)\,(2\,b + a + 1)}=$
 
$=\frac{2\,(a+b+1)^2}{a^2+b^2+3\,ab+3\,a+3\,b+1}$
 
Việc đơn giản là chứng minh:
 
$\frac{2\,(a+b+1)^2}{a^2+b^2+3\,ab+3\,a+3\,b+1}\geqq\frac{3}{2}$
 
hay:
 
$a^2+b^2+1-ab-a-b\geqq0$
 
hay:
 
$\Delta _{a}= -3\,(b- 1)^{2}\leqq 0$
 
Ai cũng biết điều này đúng!


#249
viet9a14124869

viet9a14124869

    Trung úy

  • Thành viên
  • 903 Bài viết

Bạn yeutoan89 sửa lại bài đi nhé, gõ tex có vấn đề , với lại bạn quên chưa đánh số thứ tự bài kìa 

 

Bài toán số 110 : Giả sử rằng a,b,c là các số thực dương thỏa mãn a+b+c=1 . Chứng minh rằng : 

$$\sqrt[3]{\frac{1}{a}-4b}+\sqrt[3]{\frac{1}{b}-4c}+\sqrt[3]{\frac{1}{c}-4a}\leq \sqrt[3]{\frac{5}{3}} .(\frac{1}{15abc}+\frac{6}{5})$$ 

 

P/S : Sửa đề rồi nha ai vô chém thử đi .....

                                                                                                                                               ____ Nguyễn Đức Việt____


Bài viết đã được chỉnh sửa nội dung bởi viet9a14124869: 29-04-2018 - 20:44

                                                                    SÓNG BẮT ĐẦU TỪ GIÓ

                                                                    GIÓ BẮT ĐẦU TỪ ĐÂU ?

                                                                    ANH CŨNG KHÔNG BIẾT NỮA 

                                                                    KHI NÀO...? TA YÊU NHAU .


#250
Khoa Linh

Khoa Linh

    Thiếu úy

  • Thành viên
  • 601 Bài viết

Bài 107:

Cho 3 số thực $a, b, c$ thỏa mãn $a+b+c=1$. Chứng minh:

$\frac{a}{{{a^2} + 1}} + \frac{b}{{{b^2} + 1}} + \frac{c}{{{c^2} + 1}} \le \frac{9}{{10}}$(Poland MO 1992)

Theo Dirichlet ta giả sử $(3b-1)(3c-1)\geq 0\Leftrightarrow b^2+c^2\leq \frac{1}{9}+\left ( b+c-\frac{1}{3} \right )^2=\frac{1}{9}+\left ( \frac{2}{3}-a \right )^2$

Ta có BĐT tương đương với: 

$\frac{a}{a^2+1}\leq \left ( \frac{1}{2}-\frac{b}{b^2+1} \right )+\left (\frac{1}{2}-\frac{c}{c^2+1} \right )-\frac{1}{10}\Leftrightarrow \frac{(b-1)^2}{b^2+1}+\frac{(c-1)^2}{c^2+1}\geq \frac{1}{5}+\frac{2a}{a^2+1}$

Mặt khác theo Cauchy - Schwarz ta có:

$\frac{(b-1)^2}{b^2+1}+\frac{(c-1)^2}{c^2+1}\geq\frac{(b+c-2)^2}{b^2+c^2+2}=\frac{(1+a)^2}{(b^2+c^2)+2}\geq \frac{(1+a)^2}{\frac{1}{9}+2+\left ( \frac{2}{3}-a \right )^2}=\frac{9(a+1)^2}{23-12a+9a^2}$

Vây ta đưa BĐT về BĐT 1 biến: 

$\frac{9(1+a)^2}{23-12a+9a^2}\geq \frac{a^2+10a+1}{5(1+a^2)}\Leftrightarrow (3a-1)^2(2a^2+2a+11)\geq 0$

Vậy ta có đpcm

p/s: Anh tr2512 xem lại hộ em nguồn bài toán bởi vì em làm 1 lần rồi và nó ghi là 1996 chứ không phải 1992. Em cảm ơn ...


$\sqrt[LOVE]{MATH}$

"If I feel unhappy, I do mathematics to become happy. If I am happy, I

 

do mathematics to keep happy" - Alfréd nyi 


#251
tr2512

tr2512

    Thượng sĩ

  • Thành viên
  • 272 Bài viết

Bài toán số 108 : Cho a,b,c là các số thực dương thỏa mãn a+b+c=1 . Với q=ab+bc+ca , chứng minh rằng : 

$$\frac{a}{(1-a)^2}+\frac{b}{(1-b)^2}+\frac{c}{(1-c)^2}\geq \frac{1}{2q(1-q)}$$

                                                                                                                                                              ____ Nguyễn Đức Việt ____

 

P/S : Yếu quá :P ....

Đúng là yếu thật :D

$\frac{a}{{{{\left( {1 - a} \right)}^2}}} + \frac{b}{{{{\left( {1 - b} \right)}^2}}} + \frac{c}{{{{\left( {1 - c} \right)}^2}}} \ge \frac{1}{{2q\left( {1 - q} \right)}}\\ \Leftrightarrow \frac{a}{{{{\left( {b + c} \right)}^2}}} + \frac{b}{{{{\left( {a + c} \right)}^2}}} + \frac{c}{{{{\left( {a + b} \right)}^2}}} \ge \frac{1}{{2q\left( {1 - q} \right)}}\\ \Leftrightarrow \frac{{a\left( {a + b + c} \right)}}{{{{\left( {b + c} \right)}^2}}} + \frac{{b\left( {a + b + c} \right)}}{{{{\left( {a + c} \right)}^2}}} + \frac{{c\left( {a + b + c} \right)}}{{{{\left( {a + b} \right)}^2}}} \ge \frac{1}{{2q\left( {1 - q} \right)}}$

$\Leftrightarrow \frac{a}{{b + c}} + \frac{b}{{a + c}} + \frac{c}{{a + b}} + \frac{{{a^2}}}{{{{\left( {b + c} \right)}^2}}} + \frac{{{b^2}}}{{{{\left( {a + c} \right)}^2}}} + \frac{{{c^2}}}{{{{\left( {a + b} \right)}^2}}} \ge \frac{1}{{2q\left( {1 - q} \right)}}$

Áp dụng bất đẳng thức C-S:

$\frac{{{a^2}}}{{{{\left( {b + c} \right)}^2}}} + \frac{{{b^2}}}{{{{\left( {a + c} \right)}^2}}} + \frac{{{c^2}}}{{{{\left( {a + b} \right)}^2}}} \ge \frac{1}{3}{\left( {\frac{a}{{b + c}} + \frac{b}{{a + c}} + \frac{c}{{a + b}}} \right)^2} \ge \frac{1}{3}\left[ {\frac{{{{\left( {a + b + c} \right)}^2}}}{{2{\rm{a}}b + 2bc + 2ca}}} \right]^2 = \frac{1}{{12{{\left( {ab + bc + ca} \right)}^2}}} = \frac{1}{{12{q^2}}}\\ \frac{a}{{b + c}} + \frac{b}{{a + c}} + \frac{c}{{a + b}} \ge \frac{{{{\left( {a + b + c} \right)}^2}}}{{2{\rm{a}}b + 2bc + 2ca}} = \frac{1}{{2q}}$

Vậy ta cần chứng minh:

$\frac{1}{{12{q^2}}} + \frac{1}{{2q}} \ge \frac{1}{{2q\left( {1 - q} \right)}}\\ \Leftrightarrow \frac{{\left( {1 - 3q} \right)\left( {2q + 1} \right)}}{{12{q^2}\left( {1 - q} \right)}} \ge 0$ (Luôn đúng do $ q \le \frac{1}{3}$(AM-GM))

Hoàn tất chứng minh.

 

P/s: Cho ai hứng thú, nếu quy đồng vế trái trực tiếp rồi chuyển về hàm $f(abc, ab+bc+ca)$ thì nó là hàm đồng biến theo $abc$ luôn :D, từ đó có thể làm mạnh hơn  :icon6:  :icon6:


Bài viết đã được chỉnh sửa nội dung bởi tr2512: 29-04-2018 - 17:02


#252
Diepnguyencva

Diepnguyencva

    Hạ sĩ

  • Thành viên
  • 74 Bài viết
Bài 111 Cho a,b,c>0 thỏa mãn abcd=1. Chứng minh:
$\sum \frac{1+a}{1+a^{2}}\leq 4$

Bài viết đã được chỉnh sửa nội dung bởi MoMo123: 01-05-2018 - 16:31


#253
phamhuy1801

phamhuy1801

    Trung sĩ

  • Thành viên
  • 181 Bài viết

Bài 103: Cho a,b,c không âm

Cmr: $a^2+b^2+c^2+3\sqrt[3]{(abc)^2}\geq 2(ab+bc+ac)$

P/s: TT2 là gì vậy. Có phải là Toán Học Tuổi Trẻ ko ?

 

$a+b+c \ge 3 \sqrt[3]{abc} \Leftrightarrow 3\sqrt[3]{(abc)^2} \ge \frac{9abc}{a+b+c}$

Do đó chỉ cần chỉ ra $a^2+b^2+c^2+\frac{9abc}{a+b+c} \ge 2(ab+bc+ca)$, đây là một dạng tương đương của Schur bậc $3$.

 

Bài $111$: Cho các số dương $a,b,c$ thỏa mãn $abc=1$. Chứng minh:

$\frac{1}{a+1}+\frac{1}{b+1}+\frac{1}{c+1}-\frac{1}{2} \ge \frac{1}{a+2}+\frac{1}{b+2}+\frac{1}{c+2} \ge \frac{a}{a^2+a+1}+\frac{b}{b^2+b+1}+\frac{c}{c^2+c+1}$



#254
MoMo123

MoMo123

    Sĩ quan

  • Điều hành viên THCS
  • 334 Bài viết

Theo Dirichlet ta giả sử $(3b-1)(3c-1)\geq 0\Leftrightarrow b^2+c^2\leq \frac{1}{9}+\left ( b+c-\frac{1}{3} \right )^2=\frac{1}{9}+\left ( \frac{2}{3}-a \right )^2$
Ta có BĐT tương đương với: 
$\frac{a}{a^2+1}\leq \left ( \frac{1}{2}-\frac{b}{b^2+1} \right )+\left (\frac{1}{2}-\frac{c}{c^2+1} \right )-\frac{1}{10}\Leftrightarrow \frac{(b-1)^2}{b^2+1}+\frac{(c-1)^2}{c^2+1}\geq \frac{1}{5}+\frac{2a}{a^2+1}$
Mặt khác theo Cauchy - Schwarz ta có:
$\frac{(b-1)^2}{b^2+1}+\frac{(c-1)^2}{c^2+1}\geq\frac{(b+c-2)^2}{b^2+c^2+2}=\frac{(1+a)^2}{(b^2+c^2)+2}\geq \frac{(1+a)^2}{\frac{1}{9}+2+\left ( \frac{2}{3}-a \right )^2}=\frac{9(a+1)^2}{23-12a+9a^2}$
Vây ta đưa BĐT về BĐT 1 biến: 
$\frac{9(1+a)^2}{23-12a+9a^2}\geq \frac{a^2+10a+1}{5(1+a^2)}\Leftrightarrow (3a-1)^2(2a^2+2a+11)\geq 0$
Vậy ta có đpcm
p/s: Anh tr2512 xem lại hộ em nguồn bài toán bởi vì em làm 1 lần rồi và nó ghi là 1996 chứ không phải 1992. Em cảm ơn ...

Bài này còn có một cách khác:
Ta chứng minh $\frac{a}{a^2+1} \leq \frac{18a}{25}+\frac{3}{50}$
$\Leftrightarrow (3a-1)^2(4a+3) \geq 0$ (đúng )
Tương tự với b,c cộng các bđt lại ta được đpcm

#255
DinhXuanHung CQB

DinhXuanHung CQB

    Trung sĩ

  • Thành viên
  • 118 Bài viết

Bài 107:

Cho 3 số thực $a, b, c$ thỏa mãn $a+b+c=1$. Chứng minh:

$\frac{a}{{{a^2} + 1}} + \frac{b}{{{b^2} + 1}} + \frac{c}{{{c^2} + 1}} \le \frac{9}{{10}}$(Poland MO 1996)

Giả sử $c\geq b\geq a$ xét nếu $4a+3\geq 0$ thì

$\frac{a}{a^2+1}-\frac{3}{10}-\frac{12(3a-1)}{50}=\frac{(3-a)(3a-1)}{10(a^2+1)}=\frac{-3(3a-1)^2(4a+3)}{50(a^2+1)}\leq 0$

=>$\frac{a}{a^2+1}\leq \frac{3}{10}+\frac{12(3a-1)}{50}$

từ $a\leq \frac{-3}{4}$ để ý nếu $b\leq 0$ thì $VT\leq \frac{c}{c^2+1}\leq \frac{1}{2}<\frac{9}{10}$

BDT tương đương $\sum \frac{2a}{a^2+1}\leq \frac{9}{5}$ <=> $\frac{(b-1)^2}{b^2+1}+\frac{(c-1)^2}{c^2+1}\geq \frac{2a}{a^2+1}+\frac{1}{5}$

Áp dụng $C-S$ $\frac{(b-1)^2}{b^2+1}+\frac{(c-1)^2}{c^2+1}\geq \frac{(b+c-2)^2}{(b+c)^2+2}=\frac{(a+1)^2}{(a-1)^2+2}\geq \frac{(a-1)^2}{4(a^2+1}$

mà $\frac{(a-1)^2}{4(a^2+1}-\frac{2a}{a^2+1}-\frac{1}{5}=\frac{1}{20}-\frac{5a}{2(a^2+1}>0$ có dpcm


Bài viết đã được chỉnh sửa nội dung bởi DinhXuanHung CQB: 01-05-2018 - 20:59

Little Homie


#256
DinhXuanHung CQB

DinhXuanHung CQB

    Trung sĩ

  • Thành viên
  • 118 Bài viết

Bài 107:

Cho 3 số thực $a, b, c$ thỏa mãn $a+b+c=1$. Chứng minh:

$\frac{a}{{{a^2} + 1}} + \frac{b}{{{b^2} + 1}} + \frac{c}{{{c^2} + 1}} \le \frac{9}{{10}}$(Poland MO 1996)

Có thể sử dụng $Dirichlet$ để đưa về BDT cuối cùng

$\frac{9(a+1)^2}{9a^2-12a+23}-\frac{2a}{a^2+1}-\frac{1}{5}=\frac{2(3a-1)^2(2a^2+2a+11)}{5(a^2+1)(9a^2-12a+23}\geq 0$


Bài viết đã được chỉnh sửa nội dung bởi DinhXuanHung CQB: 01-05-2018 - 21:03

Little Homie


#257
MoMo123

MoMo123

    Sĩ quan

  • Điều hành viên THCS
  • 334 Bài viết

Em cũng nghĩ như vây, mấy ngày nay online được ít, vào thấy mọi người giải bài hăng say nên rất vui nhưng nhìn lại thì thấy có những bài rất quá sức, đây chỉ mới là bắt đầu, nên đi từ dễ đến khó. Mình xin đề xuất một vài bài

$\boxed{\text{Bài 94}}$: Cho m,n,p >0 Chứng minh rằng:     $ \sum \frac{1}{m^3+1} \geq \frac{3}{mnp+1}$

$\boxed{\text{Bài 95}}$ $ Cho x,y,z \neq 0 , xyz=1$. Chứng minh rằng

$P=\frac{x^2}{(x-1)^2}+\frac{y^2}{(y-1)^2}+\frac{z^2}{(z-1)^2} \geq 1$

$\boxed{\text{Bài 96}}$ Cho $a,b,c>0$. Chứng minh 

$ (\frac{a}{a+2b})^2+(\frac{b}{b+4c})^2+(\frac{c}{c+8a})^2 \geq \frac{3}{25}$

I'm back

$\boxed{\text{Bài 95}}$ Đặt ($x,y,z)=(\frac{a^2}{bc},\frac{b^2}{ac},\frac{c^2}{ab})$

Bất đẳng thức $\Leftrightarrow $ 

$P= \frac{a^4}{(a^2-bc)^2}+\frac{b^4}{(b^2-ac)^2}+\frac{c^4}{(c^2-ab)^2} \geq 1$

Áp dụng $Cauchy Schwarzt, ta có 

$P\geq \frac{(a^2+b^2+c^2)^2}{(a^2-bc)^2+(b^2-ac)^2+(c^2-ab)^2} \geq 1$

$\Leftrightarrow (ab+bc+ca)^2 \geq 0$(đúng )

Dấu bằng xảy ra $\Leftrightarrow ab+bc+ca =0$

$\boxed{\text{Bài 96}} $

Đặt  $b=2x; c=2y ; a=z$ Ta có BĐT cần chứng minh $\Leftrightarrow $

$ P= \frac{z^2}{(z+4x)^2}+\frac{x^2}{(x+4y)^2}+\frac{y^2}{(y+4z)^2} \geq \frac{3}{25}$

Ta có:$ \frac{z^2}{(z+4x)^2}+\frac{1}{25} \geq \frac{2z}{5(z+4x)}$

Tương tự $\Rightarrow P \geq \frac{2}{5}\sum\frac{z}{z+4x} \geq \frac{2}{5}\frac{(\sum z)^2}{\sum x^2+4\sum xy} \geq \frac{3}{25}$(đpcm)


Bài viết đã được chỉnh sửa nội dung bởi MoMo123: 01-05-2018 - 21:10


#258
quynhanhlh7

quynhanhlh7

    Binh nhất

  • Thành viên mới
  • 21 Bài viết

Bài 112 : Cho ba số thực dương a,b,c thỏa mãn: $\frac{1}{a}$ + $\frac{2}{b}$ + $\frac{3}{c} = 3$ 

Chứng minh rằng: $\frac{27a^{2}}{c(c^{2}+9a^{2})}$ + $\frac{b^{2}}{a(4a^{2}+b^{2})}$ + $\frac{8c^{2}}{b(9b^{2} +4c^{2})} \geq \frac{3}{2}$



#259
minhhuy14022003

minhhuy14022003

    Binh nhất

  • Thành viên mới
  • 48 Bài viết

Bài 112 : Cho ba số thực dương a,b,c thỏa mãn: $\frac{1}{a}$ + $\frac{2}{b}$ + $\frac{3}{c} = 3$ 

Chứng minh rằng: $\frac{27a^{2}}{c(c^{2}+9a^{2})}$ + $\frac{b^{2}}{a(4a^{2}+b^{2})}$ + $\frac{8c^{2}}{b(9b^{2} +4c^{2})} \geq \frac{3}{2}$

Đặt $x=\frac{1}{a};y=\frac{2}{b};z=\frac{3}{c}$ ta có bđt tương đương với $\sum \frac{x^{3}}{x^{2}+y^{2}}\geq \frac{3}{2}$

 lại có $\frac{x^{3}}{x^{2}+y^{2}}=x-\frac{xy^{2}}{x^{2}+y^{2}}\geq x-\frac{xy^{2}}{2xy}=x-\frac{y}{2}$

tương tụ với 2 cái còn lại rồi cộng vào ta được điều phải chứng minh bạn nhé

 

 P/S: ngẫm cả topic làm dc mỗi bài này !tự thấy kém BĐT quá!



#260
MoMo123

MoMo123

    Sĩ quan

  • Điều hành viên THCS
  • 334 Bài viết

Khuấy đảo lại TOPIC nào: 

$\boxed{\text{Bài 113}}:$ Với a, b là các số thực dương, tìm giá trị lớn nhất của biểu thức 

$ M=(a+b)(\frac{1}{a^3+b}+\frac{1}{b^3+a})-\frac{1}{ab}$

$\boxed{\text{Bài 114}}$ Cho các số thực $a,b,c$ dương thỏa mãn 

$\frac{1}{a^{2}}+\frac{1}{b^{2}}+\frac{1}{c^{2}}=3 $

Tìm Max của biểu thức

$ P=\sum \frac{1}{(2a+b+c)^2}$

$\boxed{\text{Bài 115}}$:

Cho các số thực dương $a,b,c$ thỏa mãn điều kiện$\left\{\begin{matrix}a\leq b\leq c & & \\ ab+bc+ca=3 & & \end{matrix}\right.$

Chứng minh rằng $ab^2c^3<4$

$\boxed{\text{Bài 116}}$ Cho $x+y+z=12$

Chứng minh rằng:

$(x^2-4x+16)(y^2-4y+16)(z^2-4z+16) \geq 4096$

$\boxed{Bài 117}$ Cho $x,y,z $ là các số thực thuộc đoạn $[0;1]$ Chứng minh 

$\sum \frac{x}{\sqrt[3]{y^3+1}} \leq \frac{3}{\sqrt[3]{xyz+1}}$

$\boxed{Bài 118}$ Cho $a,b,c>0$ thỏa mãn $ a^2+b^2+c^2=abc=4$

Chứng minh rằng  $ a+b+c \geq \sqrt{a}+\sqrt{b}+\sqrt{c}$







Được gắn nhãn với một hoặc nhiều trong số những từ khóa sau: bất đẳng thức, holder, cosi, bunhiacopxki

1 người đang xem chủ đề

0 thành viên, 1 khách, 0 thành viên ẩn danh